inequality

This topic has expert replies
Junior | Next Rank: 30 Posts
Posts: 11
Joined: Fri Nov 08, 2013 1:25 am

inequality

by aishwarya garg » Mon Jun 30, 2014 6:06 am
Material A costs $3 per kilogram, and Material B
costs $5 per kilogram. If 10 kilograms of Material K
consists of x kilograms of Material A and y kilograms
of Material B, is x > y ?
(1) y > 4
(2) The cost of the 10 kilograms of Material K is
less than $40.

I didnt understand the explanation given in the OG. Help!

User avatar
Legendary Member
Posts: 1100
Joined: Sat May 10, 2014 11:34 pm
Location: New Delhi, India
Thanked: 205 times
Followed by:24 members

by GMATinsight » Mon Jun 30, 2014 6:35 am
Material A costs $3 per kilogram, and Material B
costs $5 per kilogram. If 10 kilograms of Material K
consists of x kilograms of Material A and y kilograms
of Material B, is x > y ?
(1) y > 4
(2) The cost of the 10 kilograms of Material K is
less than $40.


Image
Last edited by GMATinsight on Mon Jun 30, 2014 7:15 am, edited 1 time in total.
"GMATinsight"Bhoopendra Singh & Sushma Jha
Most Comprehensive and Affordable Video Course 2000+ CONCEPT Videos and Video Solutions
Whatsapp/Mobile: +91-9999687183 l [email protected]
Contact for One-on-One FREE ONLINE DEMO Class Call/e-mail
Most Efficient and affordable One-On-One Private tutoring fee - US$40-50 per hour

User avatar
Legendary Member
Posts: 1100
Joined: Sat May 10, 2014 11:34 pm
Location: New Delhi, India
Thanked: 205 times
Followed by:24 members

by GMATinsight » Mon Jun 30, 2014 6:47 am
Hi Aishwarya,

Usually people get confused in understanding the second Statement which gives
3x+5y < 40

If we consider
3x+5y = 40

If you try to find it's number of solutions then you get it only for
x = y = 5

Therefore for 3x+4y to be less than 40, x must be greater than y because 3x has more impact on total cost which is bringing it lower than $40

Which explains that answer to the question "Is x>y?" will be "YES"

Hence sufficient therefore correct [spoiler]option B[/spoiler]
"GMATinsight"Bhoopendra Singh & Sushma Jha
Most Comprehensive and Affordable Video Course 2000+ CONCEPT Videos and Video Solutions
Whatsapp/Mobile: +91-9999687183 l [email protected]
Contact for One-on-One FREE ONLINE DEMO Class Call/e-mail
Most Efficient and affordable One-On-One Private tutoring fee - US$40-50 per hour

Junior | Next Rank: 30 Posts
Posts: 11
Joined: Fri Nov 08, 2013 1:25 am

by aishwarya garg » Mon Jun 30, 2014 7:21 am
Thanks a ton! The written method was very helpful.

User avatar
Legendary Member
Posts: 1100
Joined: Sat May 10, 2014 11:34 pm
Location: New Delhi, India
Thanked: 205 times
Followed by:24 members

by GMATinsight » Mon Jun 30, 2014 7:23 am
Thank you Aishwarya... This is how I teach online... :) :) :)
"GMATinsight"Bhoopendra Singh & Sushma Jha
Most Comprehensive and Affordable Video Course 2000+ CONCEPT Videos and Video Solutions
Whatsapp/Mobile: +91-9999687183 l [email protected]
Contact for One-on-One FREE ONLINE DEMO Class Call/e-mail
Most Efficient and affordable One-On-One Private tutoring fee - US$40-50 per hour

User avatar
GMAT Instructor
Posts: 15539
Joined: Tue May 25, 2010 12:04 pm
Location: New York, NY
Thanked: 13060 times
Followed by:1906 members
GMAT Score:790

by GMATGuruNY » Mon Jun 30, 2014 10:09 am
Material A costs $3 per kilogram, and material B costs $5 per kilogram. If 10 kilograms
of material K consists of x kilograms of material A and y kilograms of material B, is x > y?
(1) y > 4
(2) The cost of the 10 kilograms of material K is less than $40.
Since 10 kilograms are purchased, x+y = 10.

Statement 1: y>4
It's possible that y=5 and x=5, in which case x=y.
It's possible that y=4.5 and x=5.5, in which case x>y.
INSUFFICIENT.

Statement 2: The cost of the 10 kilograms of material K is less than $40.
If 10 kilograms are purchased at a total cost of $40, the average cost per kilogram = 40/10 = $4.
Since the total cost here is actually LESS than $40, the average cost per kilogram is less than $4 -- LESS THAN HALFWAY between x ($3) and y ($5).
For the average cost to be LESS THAN HALFWAY between x and y, more of x must be purchased, implying that x>y.
SUFFICIENT.

The correct answer is B.
Private tutor exclusively for the GMAT and GRE, with over 20 years of experience.
Followed here and elsewhere by over 1900 test-takers.
I have worked with students based in the US, Australia, Taiwan, China, Tajikistan, Kuwait, Saudi Arabia -- a long list of countries.
My students have been admitted to HBS, CBS, Tuck, Yale, Stern, Fuqua -- a long list of top programs.

As a tutor, I don't simply teach you how I would approach problems.
I unlock the best way for YOU to solve problems.

For more information, please email me (Mitch Hunt) at [email protected].
Student Review #1
Student Review #2
Student Review #3

GMAT/MBA Expert

User avatar
GMAT Instructor
Posts: 16207
Joined: Mon Dec 08, 2008 6:26 pm
Location: Vancouver, BC
Thanked: 5254 times
Followed by:1268 members
GMAT Score:770

by Brent@GMATPrepNow » Mon Jun 30, 2014 11:21 am
Students should take notice that, for statement 2, Mitch is using an important concept regarding weighted averages to conclude that this statement is sufficient WITHOUT resorting to lengthy calculations.

If anyone is interested, we have a free and comprehensive video that covers this concept (and others) when it comes to questions involving weighted averages: https://www.gmatprepnow.com/module/gmat- ... ics?id=805

Cheers,
Brent
Brent Hanneson - Creator of GMATPrepNow.com
Image

Senior | Next Rank: 100 Posts
Posts: 31
Joined: Thu Jun 04, 2009 12:01 am
Thanked: 1 times

by unknown13 » Mon Jun 30, 2014 10:09 pm
Hi
IMO the answer is B
i also used the method
3x+4y=40 and started with x=4 and y=6

thanks and regards